Wednesday 31 July 2019

thermodynamics - Relation between a Quasistatic and a reversible process


Why is it that if a process is reversible, it is quasi-static? Does it mean that then the process is also non-dissipative if it is quasistatic?



Answer



A reversible process is defined as an ideal process, without friction, losses and entropy production. In general, such an ideal system can be represented most closely by a process with very low velocity. For example, compressing a gas piston has to be done very slowly to minimize losses and approach an ideal reversible system. In practice however, this is virtually impossible, and losses (even if very small losses) are inevitable.


Tuesday 30 July 2019

quantum mechanics - The Lorentz Transformations in the Micro-World


Two particles[or micro-observers] A and B are in relative uniform motion wrt each other in the x-x’ direction. The “observer” A decides to deduce[or interpret] the Lorentz Transformations wrt to B. Accurate knowledge of the position of B makes its[that of B] momentum highly uncertain.[$\Delta x \Delta p\ge h/2\pi$]



How does the observer A go about his job? Rather how do we interpret the Lorentz Transformation in the micro-world?




electromagnetism - Can we prove "Lumped element model" mathematically?


Maybe this question is both in the fields of engineering and physics.


As it seems the electrical quantities like Resistance, Capacitance, Inductance, and so on are quantities we assign to distributed bodies(like solid cylinders, solid cubes, etc). But later on by assuming something named "Lumped element model of electrical components" to be correct, we easily localize these quantities to some single points and then easily draw schematic models of electrical systems with zero diameter lines as wires and pointy entities as resistors or capacitors and then it turns out the calculations are always correct.


My question is:


Is there any mathematical proof to this so called model? I mean how can we assume that for example the resistance of a solid disk is located at a point on (say) its center?



P.S. I think the proof should be in a manner like how we prove the forces exerted on a not rotating rigid body could be considered as just exerted on a point particle at the body's center of mass which has a mass equal to the body's total mass.



Answer



The lumped elements approximation of electrical circuits uses the quasi-stationary approximation for the solution of Maxwell's equations. This means that the speed of electromagnetic field propagation c can be neglected (can be assumed to be infinite). Roughly this means that the dimensions $l$ of the circuit are much smaller than the vacuum wave length $l≪\lambda = c/f$ at the considered frequencies $f$.


A mathematical proof based on retarded potentials solutions of Maxwell's equations can be found, e.g., in chapter 4 of the textbook Ramo, Whinnery, van Duzer, "Fields and Waves in Communication Electronics, John Wiley & Sons Inc., 1994


electricity - Why doesn't an electron accelerate in a circuit?


Why don't electrons accelerate when a voltage is applied between two points in in a circuit? All the textbooks I've referred conveyed the meaning that when an electron traveled from negative potential to positive potential, the velocity of the electron is a constant.


Please explain.




forces - What is pressure?



When do we use the word pressure instead of force? I think pressure and force are same. Please explain without including the formula. In the example of nail penetrating into the wall whereas if we invert the nail and then it will be hard to penetrate this example doesn't help me. I think here there is only requirement of force nothing else.



Answer



I'll give you a short but better example.



Consider a plain circle made of clay. It will have a certain weight, whatever it is. Weight is the force that pushes it downwards.


If you put that circle on a water surface, it might float.


Now, take the circle and convert it into a sphere with your hands. Now the sphere does sink.


You will agree that the force is the same, because there is the same amount of mass. The mass has only been redistributed, but the amount is the same, so the weight is the same, same force.


However, now it sinks, because that force is being divided in a much smaller area.


The force is the same in both cases. The pressure is bigger in the second case, because pressure is the ratio of force divided by area.


Monday 29 July 2019

general relativity - Time Dilation Effects from simply being on a spinning planet orbiting a star in a rotating galaxy in an expanding universe.


I am a layman, so take this with a grain of salt.



I saw a TV show the other day which showed a Russian Cosmonaut who had spent more time in space than any other human. The relativistic effects of the low gravity and extreme speeds at which he had spent a decent part of his life had pushed him a small, but surprisingly non-trivial fraction of a second into the "future" as compared to the rest of us observers here on earth. I want to say it was something like a 50th of a second.


What, if any, are the relativistic effects all of us experience in an average lifespan simply by being on the earth as it travels through space in orbit and as the galaxy rotates and the universe expands.


By effects I mean as compared to a hypothetical observer who is able to remain completely motionless in space? Is it even measurable?


I know gravity is not being taken into account, so is this question even answerable?


Thanks for your patience with my question.



Answer



As the comments say, you have to be precise about your reference point when you talk about time dilation. Time dilation is always relative to something else.


But there is an obvious interpretation to your question. Suppose you have an observer well outside the Solar system and stationary with respect to the Sun. For that observer your clock on Earth is ticking slowly for two reasons:





  1. you're in a gravitional well so there is gravitational time dilation.




  2. you're on the Earth which is hurtling round the Sun at about (it varies with position in the orbit) 30 km/sec. The Earth's surface is also moving as the Earth rotates, but the maximum velocity (at the equator) is only 0.46 km/sec so it's small compared to the orbital velocity and we'll ignore it.




As it happens the problem of combined gravitational and Lotentz time dilation has been treated in the question How does time dilate in a gravitational field having a relative velocity of v with the field?, but this has some heavy maths so let's do a simplified calculation here.


The gravitational time dilation, i.e. the factor that time slows relative to the observer outside the Solar System is:


$$ \frac{t}{t_0} = \sqrt{1 - \frac{2GM}{rc^2}} $$


where $M$ is the mass of the object and $r$ is the distance from it.



For the Sun $M = 1.9891 \times 10^{30}$ kilograms and $r$ (the orbital radius of the Earth) $\approx 1.5 \times 10^{11}$ so the time dilation factor is $0.99999999017$.


For the Earth $M = 5.97219 \times 10^{24}$ kilograms and $r$ (the radius of the Earth) $\approx 6.4 \times 10^{6}$ so the time dilation factor is $0.999999999305$.


The Lorentz factor due to the Earth's orbital motion is:


$$\begin{align} \frac{1}{\gamma} &= \sqrt{1 - v^2/c^2} \\ &= 0.999999995 \end{align}$$


And to a first approximation we can simply multiply all these factors together to get the total time dilation factor:


$$ \frac{t}{t_0} = 0.999999984 $$


To put this into context, in a lifetime of three score and ten you on Earth would age about 34 seconds less than the observer watching from outside.


quantum field theory - Range Of An Interaction


Why is the Compton wavelength $\lambda_c=\frac{\hbar}{mc}$ used as a sensible measure for the range of an interaction, where $m$ is the mass of the corresponding mediator?



Answer




For massive force carriers, one finds (in natural units) an exponential dependence $\mathrm{e}^{-mr}$ that prevents long-range forces with massive mediators. Restoring SI units, one sees that the compton wavelength is the length at which the damping is exactly $\mathrm{e}^{-1}$.


homework and exercises - "Suicide Burn" rocket problem



THIS IS NOT HOMEWORK.


I'm actually in Physics I atm, and am an avid player of Kerbal Space Program. I hope to do my CS masters thesis several years from now on something space related. But none of that matters.


You're in a rocket, and one moment ago you were orbiting a nice moon (no atmosphere), magically all your horizontal velocity is gone and you are now falling directly towards the moon at the rate of g (which is technically a function of altitude).


Your rocket magically points straight up (with the engine pointing down) for you, at some point you know you want to fire the engine so you can land within some acceptable threshold so your ship doesn't explode on impact killing you.


But, you have no idea how much fuel you have, so you want to use the least amount of fuel possible, and you're smart enough to know that the longer you spend above the planet, the more fuel you're going to waste, so you want to start burning at the last second, leaving just enough time for you to get your downward velocity within that near zero margin so you don't die.



This is known as a "suicide burn."


Problem is, the moment you start burning (let's assume you either burn 100% or 0%), you immediately start increasing the time you spend above the planet, which changes when you should have started.


Let's also assume that your ship has a max thrust, which is greater than g but definitely not infinite.


So to be clear, we're looking for a time to start burning such that you land no harder than x m/s.


My question is, how would I approach such a problem?


I asked my physics professor this question and he said he'd "think on it" so I decided to ask the Internet, thanks!



Answer



I will will assume that you fall perfectly vertical and that the celestial body does not rotate.


If you assume that the mass of the rocket stays constant, then you can find when to start with the suicide burn using time reversal. Namely you "start" st the surface with your desired final velocity, $v_f$, and thrust upwards until your (specific) orbital energy matches that of your current trajectory. You can simulate this as a function of time, however in this case it will be easier to use conservation of energy,


$$ \frac{v_f^2}{2} - \frac{\mu}{R_f} + \frac{Fh}{m} = \frac{v_i^2}{2} - \frac{\mu}{R_i}, $$



where $\mu$ is the gravitational parameter of the celestial body, $R_f$ the final radius (of the surface) relative to the center of mass of the celestial body, $h$ the altitude above the surface at which you have to start the suicide burn, $F$ the amount of thrust the rocket can provide, $m$ the mass of the rocket, $v_i$ the initial velocity of the rocket and $R_i$ the initial radius of the rocket relative to the center of mass of the celestial body.


In order to find the time, $T$, it takes to perform this burn you will have to calculate the integral,


$$ T = \int_0^h \left(v_f^2-\frac{2\mu x}{R_f(R_f+x)}+\frac{2Fx}{m}\right)^{-1/2}dx, $$


however there is no general solution for this, so it will have to be calculated numerically. The used $\Delta v$ can be found with $\Delta v = \frac{F}{m}T$.


If you do want to take into account the variable mass of the rocket then you can also use time reversal and "start" at your final radius and velocity and go back in time, until your specific orbital energy matched that of your initial trajectory. For this you will have to initially guess what your final mass will be, such that at the start of your burn your total mass is equal to $m$.


specific reference - Introduction to neutron star physics


I enjoy thinking about theoretical astrophysics because I want to understand black holes. Given that no one understands black holes, I like to ponder the nearest thing to a black hole: a neutron star! I have searched around the web for pedagogical discussions of the structure of neutron stars such as this link from NASA: http://heasarc.nasa.gov/docs/objects/binaries/neutron_star_structure.html, but none seems to be at an advanced enough level for my liking. The problem is that I do not know what literature I should read in order to learn more.


What is the current state of neutron star research? What are some good review articles?


More specifically, I am curious about theoretical predictions for "starquakes" referenced in the link above, and how they would look to an observer on Earth. I would also be interested in understanding what happens to gas falling onto a neutron star -- specifically, if Sol was spiraling to its death by a neutron star.




Answer



There are a few standard textbooks on neutron star.


For interior structure and nuclear physics side two books by Glendenning are good.


http://www-nsdth.lbl.gov/~nkg/description.html


For more general relativity side Shapiro and Teukolsky has been a standard texk book for many years.


http://www.amazon.com/Black-Holes-White-Dwarfs-Neutron/dp/0471873160


Finally, if you seek for real rigor, a new book by Friedman & Stergioulas is must.


http://www.amazon.com/Rotating-Relativistic-Stergioulas-Cambridge-Monographs/dp/0521872545


There are several review papers including two in Living Review.


http://relativity.livingreviews.org/Articles/lrr-2003-3/



http://relativity.livingreviews.org/Articles/lrr-2007-1/


Several by Lattimer and Prakash are also good starting point. For example,


http://arxiv.org/abs/astro-ph/0612440


Sunday 28 July 2019

Formula for scattering and energy change of photons on (naked) nuclei


What is the formula for scattering and energy change of photons on (naked) nuclei?


(On wikipedia Compton scattering does only explain scattering of photons by electrons, and I'm not even aware if the scattering with atomic nuclei has a different name, I always assumed it would be Compton scattering, too.)


This is a followup from X-ray shielding .




thermodynamics - Symbols of derivatives


What is the exact use of the symbols $\partial$, $\delta$ and $\mathrm{d}$ in derivatives in physics? How are they different and when are they used? It would be nice to get that settled once and for all.



$$\frac{\partial y}{\partial x}, \frac{\delta y}{\delta x}, \frac{\mathrm{d} y}{\mathrm{d} x}$$



  • For what I know, $\mathrm{d}$ is used as a small infinitisemal change (and I guess the straight-up letter $\mathrm{d}$ is usual notation instead of italic $d$, simply to tell the difference from a variable).

  • Of course we also have the big delta $\Delta$ to describe a finite (non-negligible) difference.

  • And I have some vague idea that $\partial$ is used for partial derivatives in case of e.g. three-dimensional variables.

  • Same goes for $\delta$, which I would have sworn was the same as $\partial$ until reading this answer on Math.SE: https://math.stackexchange.com/q/317338/


Then to make the confusion total I noticed an equation like $\delta Q=\mathrm{d}U+\delta W$ and read in a physics text book that:



The fact that the amount of heat [added between two states] is dependent on the path is indicated by the symbol $\delta$...




So it seems $\delta$ means something more? The text book continues and says that:



a function [like the change in internal energy] is called a state function and its change is indicated by the symbol $\mathrm{d}$...



Here I am unsure of exactly why a $\mathrm d$ refers to a state function.


So to sum it up: down to the bone of it, what is $\delta$, $\partial$ and $\mathrm{d}$ exactly, when we are talking derivatives in physics.


Addition


Especially when reading a mathematical process on a physical equation like this procedure:


$$\delta Q=\mathrm{d}U+p\mathrm{d}V \Rightarrow\\ Q=\Delta U+\int_1^2 p \mathrm{d}V$$



It appears that $\delta$ and $\mathrm{d}$ are the same thing. An integral operation handles it the same way apparently?



Answer



Typically:



  • $\rm d$ denotes the total derivative (sometimes called the exact differential):$$\frac{{\rm d}}{{\rm d}t}f(x,t)=\frac{\partial f}{\partial t}+\frac{\partial f}{\partial x}\frac{{\rm d}x}{{\rm d}t}$$This is also sometimes denoted via $$\frac{Df}{Dt},\,D_tf$$

  • $\partial$ represents the partial derivative (derivative of $f(x,y)$ with respect to $x$ at constant $y$). This is sometimes denoted by $$f_{,x},\,f_x,\,\partial_xf$$

  • $\delta$ is for small changes of a variable, for example minimizing the action $$\delta S=0$$ For larger differences, one uses $\Delta$, e.g.: $$\Delta y=y_2-y_1$$


NB: These definitions are not necessarily uniform across all subfields of physics, so take care to note the authors intent. Some counter-examples (out of many more):




  • $D$ can denote the directional derivative of a multivariate function $f$ in the direction of $\mathbf{v}$: $$D_\mathbf{v}f(\mathbf{x}) = \nabla_\mathbf{v}f(\mathbf{x}) = \mathbf{v} \cdot \frac{\partial f(\mathbf{x})}{\partial\mathbf{x}}$$

  • More generally $D_tT$ can be used to denote the covariant derivative of a tensor field $T$ along a curve $\gamma(t)$: $$D_tT=\nabla_{\dot\gamma(t)}T $$

  • $\delta$ can also represent the functional derivative: $$\delta F(\rho,\phi)=\int\frac{\delta F}{\delta\rho}(x)\delta\rho(x)\,dx$$

  • The symbol $\mathrm{d}$ may denote the exterior derivative, which acts on differential forms; on a $p$-form, $$\mathrm{d} \omega_p = \frac{1}{p!} \partial_{[a} \omega_{a_1 \dots a_p]} \mathrm{d}x^a \wedge \mathrm{d}x^{a_1} \wedge \dots \wedge \mathrm{d}x^{a_p}$$ which maps it to a $(p+1)$-form, though combinatorial factors may vary based on convention.

  • The $\delta$ symbol can also denote the inexact differential, which is found in your thermodynamics relation$${\rm d}U=\delta Q-\delta W$$ This relation shows that the change of energy $\Delta U$ is path-independent (only dependent on end points of integration) while the changes in heat and work $\Delta Q=\int\delta Q$ and $\Delta W=\int\delta W$ are path-dependent because they are not state functions.


thermodynamics - How do I set up the tridiagonal matrix for a heat diffusion with layers of different thermal diffusivity?


I have Scala code that recreates the Crank-Nicolson solutions for the diffusion equations, and matches 'Excel for Scientists and Engineers' (Joe Billo, Wiley).


However, I would like to be able to replicate the results from H. Asan's 'Numerical computation of time lags and decrement factors' (including the decrement factor with very thin materials) and also the multi-layer positioning results from the BS2013 paper 'Optimizing insulation-thermal mass layer distribution from maximum time lag and minimum decrement factor point of view'.


Both papers suggest that Crank-Nicolson is at the heart of the approach, however there is a complexity with the treatment of energy flow at the outer and inner surface, and with different material layers, that are not covered in the simple diffusion treatments that I found.


How do I set up the matrix - or how do I connect a series of such matrices together?


I had thought that perhaps a simple treatment would give a given wall thickness a behavior like a delay factor and an amplitude scaling - but it does not seem to be possible to do this and sum the phase delays and multiply the decrements, because the results above suggest that the ordering is not transitive.



How can I proceed - given that I'm a lot better at coding than solving PDEs.




thermodynamics - Mathematical proof of non-negative change of entropy $Delta Sgeq0$


I understand that we can prove that for any process that occurs in an isolated and closed system it must hold that


$$\Delta S\geq0$$


via Clausius' theorem. My question is, how can I prove this in a mathematical way?



Answer



In the context of quantum mechanics, the entropy of a system whose initial state is given by a density matrix $\rho(0)$ is given by the so-called von Neumann entropy; $$ S_\mathrm{vn}(\rho) = -k\,\mathrm{tr}(\rho\ln\rho) $$ For an isolated system, quantum mechanical time evolution is unitary; for each time $t$, there is a unitary operator $U(t)$ such that the state of the system at time $t$ is given by $$ \rho(t) = U(t) \rho(0)\, U^\dagger (t) $$ It can be shown that the von Neumann entropy is invariant under unitary similarity transformation of $\rho$; in other words $$ S_\mathrm{vn}(U\rho U^\dagger) = S_\mathrm{vn}(\rho) $$ and it immediately follows that $$ S_\mathrm{vn}(\rho(0)) = S_\mathrm{vn}(\rho(t)) $$ In other words, the entropy of an isolated quantum system does not change with time in accordance with the second law of thermodynamics.


Author's Admission. I have always been somewhat bothered by the argument I just gave you, not because I think it's incorrect, but rather because in light of the conclusion we draw from it regarding isolated systems, why don't people say that the stronger statement $dS=0$ for isolated systems as opposed to $dS\geq 0$. It's not that these are inconsistent statements; one is just stronger than than the other, so I would think one should simply assert the stronger one in the context of isolated systems.



Addendum. In response to my "admission," I should note that there is a cute argument I have seen for the non-negativity of a change in total (von-Neumann) entropy of an isolated system provided one defines total entropy properly. Here it is.


Suppose that we have an isolated system, let's call it the universe, described by a Hilbert space $\mathcal H$. Suppose that this system can be divided into two subsystems $a$ and $b$ so that the combined Hilbert space can be written $\mathcal H = \mathcal H_a\otimes\mathcal H_b$. If the density matrix of the universe is $\rho$, then the density matrices of the subsystems $a$ and $b$ are defined as partial traces over $\rho$; $$ \rho_a = \mathrm{tr}_{\mathcal H_a}\rho, \qquad \rho_b = \mathrm{tr}_{\mathcal H_b}\rho $$ Now we can prove the following:



If systems $a$ and $b$ are initially uncorrelated, then then the total entropy $S(\rho_a) + S(\rho_b)$ will never be lower than at the initial time.



Proof. If the systems are initially uncorrelated, then by definition the total density operator at the initial time is a tensor product $\rho(0) = \rho_a^0\otimes \rho_b^0$. It follows from taking partial traces and using the fact that the density operator is unit trace that the density matrices of the subsystems $a$ and $b$ at the initial time are $$ \rho_a(0) = \rho_a^0, \qquad \rho_b(0) = \rho_b^0 $$ Now, at any later time, the total density matrix evolves unitarily, so that $$ S(\rho(0)) = S(\rho(t)) $$ On the other hand, entropy is subadditive which means that $$ S(\rho(t)) \leq S(\rho_a(t))+S(\rho_b(t)) $$ and is additive for uncorrelated systems which gives $$ S(\rho(0)) = S(\rho_a(0)) + S(\rho_b(0)) $$ Putting this all together yields $$ S(\rho_a(0)) + S(\rho_b(0)) \leq S(\rho_a(t))+S(\rho_b(t)) $$


I've always been somewhat unsatisfied with this argument, however, because (i) it assumes that the subsystems are originally uncorrelated and (ii) it's not clear to me that the definition of total entropy as the sum of the entropies of the reduced density operators of the subsystems is what we should be calling $S$ when we write $\Delta S \geq 0$.


By the way, this argument was stolen from lectures I took: Eric D'Hoker's quantum lecture notes.


Saturday 27 July 2019

homework and exercises - What is going on in the system? How are the formulas $mg sin(x)$ and $mg cos(x)$ derived?


When a load is resting on an inclined plane, there is force $mg \sin(x)$ that's vertical to the inclined plane and force $mg \cos(x)$ horizontal to the plane acting on it like this:


enter image description here


My textbook says the weight $mg$ has two components: $mg \sin(x)$ and $mg \cos(x)$.


But when adding up $mg \sin(x)$ to $mg cos(x)$ it does not equal to $mg$.


If mg and N are the only forces in this system, which means the net force should be 0, how should I explain the existence of $mg \sin(x)$ and $mg \cos(x)$? Does $mg \sin(x)$ and $mg \cos(x)$ have their own reaction force to cancel them out and turn the net force to be 0 too or is it wrong the say the system's net force is 0?



And how are the formulas $mg \sin(x)$ and $mg \cos(x)$ that are used to describe these two forces derived?



Answer



Because $mg\cos x$ and $mg\sin x$ are orthogonal vectors, not colinear, and the norm of their sum is the Pythagorean sum. This kind of addition is pretty common in physics as well as in other places basic vector algebra shows up -- an elegant example is in statistics, where independent (orthogonal) random variables get added in a Pythagorean way, while multiple recordings of a variable get added up linearly.


The fact that we can do this is actually a very good indicator -- and part of the proof -- of the fact that physical forces are mathematical vectors, i.e. that it satisfies vector algebra. So the answer to the question is really an empirical one -- this is how forces get added, and can be mathematically explained as them being vectors.


newtonian mechanics - What is the third law action pair of buoyancy?


Consider a block floating in a fluid. The force due to gravity balances the buoyant force exerted on the block. But there must be a third-law action pair to the buoyant force. If the block exerts a force on the fluid, that would mean that the pressure in fluid just below the block is more than that in other places that are on the same horizontal level.


enter image description here


I do not believe that to be the case, since it doesn't conform to real world observations.


Also, if the block has a $m$ and density $\rho$, and the fluid + the container has a mass $M$. What is the mass of the block + container + fluid (as in the picture), as measured by a weighing machine underneath the container.


According to my common sense, since the buoyant force is internal, the measured mass of the system should be $m + M$. Am I right?



Answer



The buoyant block does exert a force on the water, it's force is equal to the mass of the displaced water, so the pressure of the water immediately beneath the block is exactly the same as the pressure of the water at that height in the rest of the container.


Indeed, the mass of the system is just the mass of container with water + mass of block


special relativity - Why is the speed of light used to define the fourth axis of spacetime?


The four axes of spacetime are $x, y, z$ and $ct$, where $c$ is the speed of light, and $t$ is time. Why is the speed of light (not any other speed) used to define the fourth axis of spacetime? If someone answers that the speed of light is the maximum speed among all speeds, such answer does not satisfy me. I would further ask: why is the maximum speed used to define the fourth axis of spacetime?




conservation laws - Does a particle annihilate only with its antiparticle? If yes, why?


Or to put the question another way - what is the result of a proton-positron collision, or an up quark-charm antiquark collision, etc.? As far as I know, annihilation happens only between particles of opposite charge and same mass, but perhaps I am wrong?


And if the types of annihilation mentioned above cannot happen, what are the reasons?



Answer



It depends on your definition of annihilation. But microscopically all processes are described by Feynman diagrams such as these


enter image description here



of which last one describes electron positron annihilation (if it weren't for the typo in the out-going photon). But as you can see it's all a simple matter of how you turn your head around and the very same diagram represents emission (or absorption) of photon by electron (or positron). Does electron annihilate with photon and create a brand new electron? You can certainly interpret it that way. In other words, it's just a question of terminology and interpretation. Actual physics doesn't depend on how you call the process. It is encoded in the underlying math of quantum field theory (QFT).


In any case, the punchline is that annihilation (in the strict sense of particle-antiparticle inelastic collision) doesn't have a special place in one's vocabulary once they learn their QFT and particle physics. It's just one particular kind of interaction. So you might as well ask which arbitrary interactions are allowed. And answer to that is: there's quite a lot of them and they are described by Standard Model. But the basic picture is that particles can be charged under certain charges: either the familiar electromagnetic, or less familiar weak and strong charges. Or in more modern language whether some families of particles form a multiplet under some gauge group. For weak force with group SU(2) you get lepton (e.g. electron-neutrino) and quark (e.g. up-down) doublets. For strong force with group SU(3) you need triplets and these are precisely the red green blue colors of quarks that you probably heard about.


In any case, for every multiplet there is a diagram like the ones above where you have two charged particles and one mediating particle between them (photon, weak bosons or gluons). Besides this you can also get funnier diagrams with e.g. three or four gluon lines. But that's it, these are all of allowed interactions of Standard Model.


Friday 26 July 2019

general relativity - What does "finite but unbounded universe" mean?


In Einstein's book about relativity, he says that his theory predicts that the shape of the universe would be finite but unbounded.


But how is this possible?



What's the difference between an infinite and an unbounded universe?


Therefore, what does it mean for a universe to be "finite but unbounded"?



Answer



I will use 2D surfaces embedded in a 3D space for this answer, because we are accustomed to imagining such surfaces in everyday life. Just remember that in General Relativity, the 'surface' we are talking about is in fact 4D space-time!


As an example of a bounded Universe, imagine a 2D disc in 3D space. Now imagine that you are confined to one side of that disc, so you are in fact confined to a 2D space.


If you walk without turning for long enough on that disc, you will eventually reach its edge. That is, since the disc has a finite surface area rather than an infinite surface area, you cannot walk for an infinitely-long time without 'falling off'.


As an example of an unbounded Universe, imagine a sphere in 3D space. Now imagine that you are confined to the surface of that sphere, so you are in fact confined to a 2D space.


You may walk around that sphere as much as you like, and never come to an edge, despite the fact that the surface of the sphere has a finite area. That is, if you walk for an infinitely long time without turning, you will keep coming back to where you started, rather than walking away to infinity.


This is also true of many other 2D surfaces, such as the surfaces of torii.


Extending the idea to 4D spacetime in an intuitive way is difficult, but one could think of a finite, unbounded Universe as one in which, if you travel long enough in the same direction in spacetime, you come back to where you began, rather than reaching the 'edge of the Universe'.



homework and exercises - Find angular momentum about any point


How do I find the angular momentum of a body about any point? We know that $L=I\omega$ for a body rotating in space, where $L$ denotes the angular momentum, $I$ denotes the moment of inertia and $\omega$ denotes the angular velocity. However, this is only applicable for fixed axis of rotation, instantaneous axis of rotation and center of mass. Can somebody state and prove the value of the angular momentum of a body about any point? (if a formula for that exists)?



Answer



Let's suppose I have some system and I know the system's total mass $M$, the system's center of mass position $\newcommand{\v}[1]{\mathbf{#1}}\v{r}_{cm}$, and the systems angular velocity $\v{L}_{cm}$, in the frame where the center of mass is the origin. How do I find $\v{L}'$, the angular momentum with respect to some other origin, say $\v{r}_{0} $, which is moving at a velocity $\v{v}_0$? That is the question I will answer.


The answer is easy to understand intuitively. The total angular momentum in the new frame is the sum of two terms. The first term is the angular momentum in the center of mass frame $\v{L}_{cm}$. This piece is intrinsic to the motion in the sense that it does not depend on the frame. The second piece is frame dependent, but has a simple form which does not depend on the details of the system. The frame dependent piece is $ M \left(\v{r}_{cm}-\v{r}_0\right) \times \left(\v{v}_{cm}-\v{v}_0\right)$. Notice this is just the angular momentum of a point particle with position $\v{r}_{cm}$ and velocity $\v{v}_{cm}$. So the system can be modeled as a point particle for the purposes of calculating the frame dependent piece.


It is not hard to prove that the angular momentum decomposes this way. To do this, we will introduce the notation $\langle X \rangle = \int X dm$, so that when we write $\langle \v{r} \rangle$, we mean $\int \v{r} dm = M \v{r}_{cm}$. With this notation, the angular momentum in the frame with origin $\v{r}_0$ moving at velocity $\v{v}_0$ is $$\begin{equation} \begin{aligned} \v{L}'=&\langle \left(\v{r}-\v{r}_0\right) \times \left(\v{v}-\v{v}_0\right)\rangle\\ =&\langle \left(\left(\v{r}-\v{r}_{cm}\right) +\left(\v{r}_{cm} -\v{r}_0\right)\right) \times \left(\left(\v{v}-\v{v}_{cm}\right)+\left(\v{v}_{cm}-\v{v}_0\right)\right)\rangle\\ =&\overbrace{\langle \left(\v{r}-\v{r}_{cm}\right) \times \left(\v{v}-\v{v}_{cm}\right) \rangle}^{\v{L}_{cm}}\\ &+\langle \left(\v{r}-\v{r}_{cm}\right) \times\left(\v{v}_{cm}-\v{v}_0\right) \rangle\\ &+\langle \left(\v{r}_{cm} -\v{r}_0\right) \times \left(\v{v}-\v{v}_{cm}\right) \rangle\\ &+\langle \left(\v{r}_{cm} -\v{r}_0\right) \times \left(\v{v}_{cm}-\v{v}_0\right) \rangle\\ =&\v{L}_{cm}\\ &+ \overbrace{\langle\v{r}-\v{r}_{cm} \rangle}^{0} \times\left(\v{v}_{cm}-\v{v}_0\right)\\ &+\left(\v{r}_{cm} -\v{r}_0\right) \times \overbrace{\langle \v{v}-\v{v}_{cm}\rangle}^{0}\\ &+M \left(\v{r}_{cm} -\v{r}_0\right) \times \left(\v{v}_{cm}-\v{v}_0\right) \\ =&\v{L}_{cm} + M \left(\v{r}_{cm} -\v{r}_0\right) \times \left(\v{v}_{cm}-\v{v}_0\right) \end{aligned} \end{equation}$$


Above, in the third line, we find that $\v{L}'$ is the sum of four terms. The first is the angular momentum in the center of mass frame, $\v{L}_{cm}$, in the second and third terms, a constant can be factored out of the angle brackets and what remains in the brackets averages to zero. In the fourth term, the quantity in brackets is just a constant, so the brackets amount to multiplication by $M$. The two surviving terms are exactly the terms described in the previous paragraph.




You might think that you use the parallel axis theorem here. The parallel axis theorem is actually a special case of this where the displacement of the origin is perpendicular to the axis of rotation, and your new origin is some point embedded in the object (assumed to be rigid). By embedded in the object, I mean that the new origin is moving at the same velocity of the object at that point so that $\v{v}_0-\v{v}_{cm}= \boldsymbol{\omega} \times \left( \v{r}_0 - \v{r}_{cm}\right)$.


The equation we derived in this answer then predicts \begin{equation} \begin{aligned} \v{L}' &= \v{L}_{cm} + M \left(\v{r}_{cm}-\v{r}_0\right) \times \left(\boldsymbol{\omega} \times \left(\v{r}_{cm} - \v{r}_0\right)\right)\\ &= \v{L}_{cm} + M\left( \boldsymbol{\omega} \left(\v{r}_{cm}-\v{r}_0\right)^2 - \left(\v{r}_{cm}-\v{r}_0\right) \overbrace{\left(\v{r}_{cm}-\v{r}_0\right)\cdot \boldsymbol{\omega}}^{0} \right)\\ &=\v{L}_{cm} + M \boldsymbol{\omega} \left(\v{r}_{cm}-\v{r}_0\right)^2. \end{aligned} \end{equation}.


On the other hand, the parallel axis theorem would tell us to make the subsitution $I_{cm} \to I_{cm} + M \left(\v{r}_{cm}-\v{r}_0\right)^2$. Thus we would have $$I_{cm} \boldsymbol{\omega} \to \left(I_{cm} + M\left(\v{r}_{cm}-\v{r}_0\right)^2\right)\boldsymbol{\omega} = I_{cm} \boldsymbol{\omega} +M \left(\v{r}_{cm}-\v{r}_0\right)^2\boldsymbol{\omega}.$$ So that $\v{L}_{cm} \to \v{L}_{cm} + M\left(\v{r}_{cm}-\v{r}_0\right)^2\boldsymbol{\omega}$. I.e., $\v{L}' = \v{L}_{cm} + M\left(\v{r}_{cm}-\v{r}_0\right)^2\boldsymbol{\omega}$. Thus we see how the answer we get are the same in this special case, and the parallel axis theorem can be used. However your question concerns more general transformations.


special relativity - Relativistic mass of photon



What is the relativistic mass of one photon, i.e. what is $N$ here: $10^{-N}$ kg?



Answer



First, photons do not have rest mass. Thus, they move in space with speed c, in vacuum, when measured locally.



Because they do not have rest mass, they do not move in the time dimension, their speed in the time dimension is 0. Thus, they are not experiencing time the way we do.


They do have energy, and it is related to their frequency. They do have stress-energy, so they do excert gravitational pressure.


They do not use the term relativistic mass any more, because in special relativity, mass is not a conserved quantity. It is not used in particle physics. It is not used anymore because it causes confusion. It connects Newtonian inertial mass with relativistic energies. It is only used for theoretical experiments with spaceships trying to reach relativistic speed and the fuel they would need.


mass - Neutrino Oscillations and Conservation of Momentum


I would like to better understand how neutrino oscillations are consistent with conservation of momentum because I'm encountering some conceptual difficulties when thinking about it. I do have a background in standard QM but only rudimentary knowledge of particle physics.


If the velocity expectation value of a neutrino in transit is constant, then it would appear to me that conservation of momentum could be violated when the flavor eigenstate at the location of the neutrino source is different from that at the location of the interaction, since they are associated with different masses.


For this reason I would think that the velocity expectation value changes in transit (for instance, in such a way to keep the momentum expectation value constant as the neutrino oscillates), but then it seems to me that the neutrino is in effect "accelerating" without a "force" acting on it (of course, since the momentum expectation value is presumed constant, there may not be a real problem here, but it still seems strange).


Any comments?





Thursday 25 July 2019

conformal field theory - Questions about classical and quantum scale invariance


This is kind of a continuation of this and this previous questions.




  • Say one has a free "classical" field theory which is scale invariant and one develops a perturbative classical solution for an interacting version of that. Then what can be the constraining effects on the perturbation theory from the fact that the free theory was scale invariant? (for an example like this see this recent paper)




  • Can one give a QFT example where this can happen that a non-marginal coupling flows to a non-zero fixed point and the anomalous dimension of the corresponding operator adjusts so that the coupling becomes marginal at that point? (..for the fixed point of the RG flow to be interpreted as a point of scale invariance this adjustment has to happen!..)







density operator - Relationship between the Lindblad Equation and Redfield Equation


Both the Lindblad and Redfield Equation both model the open quantum system dynamics given a Hamiltonian and some operators. What is the relationship between the two equations? How can they transformed from Lindblad to Redfield, given the Hamiltonian, and Lindblad operators? Is this not possible?




electromagnetism - Why does the vacuum permeability have the value of $pi$ in it?


The vacuum permeability, or the capability of the vacuum to permit magnetic field lines, contains the value of $\pi$. Why? What does this have to do with the ratio of a circle's circumference to its diameter?



Answer



This is nothing but a choice of units.


Let me make that (hopefully) more clear by explaining more about how choosing units in electromagnetism works:





  • Coulomb's Law is $\vec F = k \frac{q_1 q_2}{r^2} \hat{\vec r}$ for the force between two charges $q_1$ and $q_2$. $k$ is different in the various systems of units - essentially, it depends on how the unit of charge is defined.




  • Ampere's Law is $\vec F = k' I_1 I_2 \oint_{C_1} \oint_{C_2} \frac{d\vec r_1 \times (d\vec r_2 \times \vec r)}{|r|^3}$ for the force between two currents along $C_1$ and $C_2$. $k'$ is different in the various systems of units - essentially, it depends on how the unit of current is defined.




In electrodynamics, we find out that $$ k / k' = c^2 $$ otherwise, we are free to choose. In CGS, we define $k = 1$, and in SI $k' = 10^{-7} \frac{Vs}{Am}$.


Now we introduce constants $$ \mu_0 = 4\pi\, k', \quad \varepsilon_0 = \frac{1}{4\pi k} $$ That is nothing more than a definition - the factors of $4\pi$ simplify some formulas later on, e.g. when fields are integrated over the surface of a sphere.


water - Why is the absolute zero a rational number in Celcius?


From the question "Why is the absolute zero -273.15ÂşC?" I understood that 1°C is the 100th part of the difference of melting and boiling temperature of water (this is my high school physics, maybe there is a more accurate definition). So we linked the value of 1°C to a physical quantity.


Then we measure another physical quantity (the absolute zero) and it turns out this is exactly 273.15°C less the melting temperature of water. So how come those two quantities have a rational ratio?


If we look at other constants (gravitational, Planck or whatever) they all are irrational, aren't they?



What did I miss? Is it because the melting point of water is not exactly 0°C (maybe something really close to 0 like 0.000565...)?



Answer




it turns out this is exactly 273.15°C less the melting temperature of water.



Actually, "Kelvin" and "degrees Celsius" are defined such that there are 273.16 degrees between absolute zero and the triple point temperature of water. Degrees Celsius are defined as $K - 273.15$.


The freezing point of water is a measured quantity and is not exactly 273.15K nor 0°C and isn't necessarily a rational number.


particle physics - About free quarks and confinement


I simply know that a single free quark does not exist. What is the reason that we can not get a free quark?


If we can't get a free quark then what is single-top-quark?



Answer



A free quark is like the free end of a rubber band. If you want to make the ends of a rubber band free you have to pull them apart, however the farther apart you pull them the more energy you have to put in. If you wanted to make the ends of the rubber band truly free you'd have to make the separation between them infinite, and that would require infinite energy. What actually happens is that the rubber band snaps and you get four ends instead of the two you started with.



Similarly, if you take two quarks and try and pull them apart the force between them is approximately independent of distance, so to pull them apart to infinity would take infinite energy. What actually happens is that at some distance the energy stored in the field between them gets high enough to create more quarks, and in stead of two separated quarks you get two pairs of quarks.


This doesn't happen when you pull apart a proton and electron because the force between them falls according to the inverse square law. The difference between the electron/proton pair and a pair of quarks is that the force between the quarks doesn't fall according to the inverse square law. Instead at sufficiently long distances it becomes roughly constant.


I don't think this is fully understood (it certainly isn't fully understood by me :-), but it's thought to be because the lines of force in the quark-quark field represent virtual gluons, and gluons attract each other. This means the lines of force collect together to form a flux tube. By contrast the electron-proton force is transmitted by virtual photons and photons do not attract each other.


Finally, top quarks are usually produced as a top anti-top pair. It is possible to create a single top quark, but it's always paired with a quark of a different type so you aren't creating a free quark.


Wednesday 24 July 2019

black holes - Is it possible to change the frequency of the light by magnetic fields?


I do not recall where, but I have read someting about the light approaching a blackhole. In the writing, it was said that an object would gain kinetical energy due to the blackhole's gravity, and would increase its speed. On the other hand, when the light approaches than an object, it would gain kinetical energy as well, but the speed of it would not change: The frequency would. And so, an idea appeared in my head; since the mass-gravity is the weakest among gravities, what would happen if it was tried to affect the frequency of light through magnetism. Of course, even though the mass-gravity is the weakest, the blackhole is being talked of, so to be equal, a lot of magnetic force would be needed, doubtly. Yet, regardless of the ammount, I would like to learn if the statement I've read about is true, and if it is; I would like to learn if it is possible to affect the light by magnetism.




differential geometry - Usefulness of Curl and Divergence as Multilinear Maps


Early in differential geometry, texts typically reformalize our usual gradient, divergence and curl operators as covariant tensors rather than vectors. This is primarily motivated by the observation that they transfer covariantly rather than contravariantly, and therefore need to be assigned a new tensorial structure to govern the transformation rules.


However, in assigning additional structure to these operators, it lends them additional interpretation that does not seem very physically useful.


Take, for example, the curl. Observing that it transforms covariantly, we can assign it a covariant tensor structure. When we want to take the curl of a vector field, we first map the vectors to covectors, take an exterior derivative, and then remap the resulting 2-forms back to vectors.


However, while this achieves our goal of defining the curl in a way that transforms covariantly, we've unintentionally given it a new interpretation to its codomain as maps from 2 vectors to a real number. And, even worse, any vector field is now a map from 2 vectors to a real number via hodge dual. While defining new structures has given us something more useful, it seems to have equivocated our familiar vectors with uninteresting structural baggage in return.


It seems like I'm forced to either, 1) find a useful interpretation of the codomain of the curl operator of the form $(a_x \ dy \wedge dz + a_y \ dx \wedge dz + a_z dx \wedge dy)(v_1, v_2)$ as a useful real number, so that I can be at peace with thinking of the curl as a map from covectors to covectors and not just a vector operator with covariant transformation rules, or 2) accept that sometimes in making physically useful things formal with tensor calculus and the hodge dual, we've created some uninteresting interpretations that simply weren't possible beforehand.


What's a physicist to do?



Answer



This answer takes into account the elaboration on the question which takes place in the comments.


Let's build a tensor space. I won't bother with the definition of a tangent vector as a "directional derivative" operator - we'll just start from the fairly intuitive notion of tangent vectors as little "arrows" which live in the tangent space at a point $p$ in some manifold $\mathcal{M}$.



Vectors


A vector $X$ defined at a point $p\in M$ can be expanded in terms of a vector basis $\hat e_i$. That is, $$X = X^i \hat e_i$$ where we follow the Einstein summation convention.


Covectors


A covector is an $\mathbb{R}-$linear map which eats vectors and returns real numbers. While the vector space is spanned by the vector basis $\hat e_i$, the covector space is spanned by the covector basis $\epsilon_i$. In principle, these bases may be chosen independently of one another, but one typically chooses the covector basis such that $$\epsilon_i(\hat e_j) = \cases{+1 & $i=j$ \\ 0 & $i\neq j$}$$


The action of a covector $\omega = \omega_i \epsilon^i$ on a vector $X = X^i \hat e_i$ is as follows: $$\omega(X) = \omega_i \epsilon^i(X^j \hat e_j) = \omega_i X^j \epsilon^i(\hat e_j) = \omega_i X^i$$


We can similarly define the action of a vector on a covector: $$X(\omega) \equiv \omega(X)$$


Therefore, we can regard vectors and covectors as objects which eat each other and return real numbers.


Tensors


A $(p,q)-$tensor $T$ is an $\mathbb{R}-$multilinear map which eats $p$ covectors and $q$ vectors and returns a real number. From this standpoint, functions on the manifold are $(0,0)-$tensors, vectors are $(1,0)-$tensors, and covectors are $(0,1)-$tensors. As an example, consider a $(1,1)-$tensor $T$. Its action on a covector $\omega$ and a vector $X$ is then $$ T(\omega,X) = T(\omega_i \epsilon^i,X^j \hat e_j) = \omega_i X^j T(\epsilon^i,\hat e_j) = \omega_i X^j T^i_{\ j}$$ where $$T^i_{\ j} \equiv T(\epsilon^i,\hat e_j)$$ are called the components of $T$ in the chosen basis.


Metric Tensors



A metric tensor $g$ is a symmetric, positive-definite $(0,2)$-tensor. Its action on two vectors $X$ and $Y$ can be written $$g(X,Y) = g(X^i\hat e_i,Y^j \hat e_j) = X^i Y^j g(\hat e_i,\hat e_j) = X^i Y^j g_{ij}$$ where the components $$g_{ij}$$ can be put in the form of a symmetric, invertible matrix. This generalizes the notion of the familiar dot product between vectors.


Flat


We can define an operation called flat which takes a vector $X$ to a covector $X^\flat$ by plugging it into the first slot of the metric but leaving the second slot empty. In other words, $$X^\flat \equiv g(X,\bullet)$$ Its action on a vector $Y$ is given by $$X^\flat(Y) = (X^\flat)_i \epsilon^i(Y^j \hat e_j) = (X^\flat)_i Y^j \epsilon^i(\hat e_j) = (X^\flat)_i Y^i$$ but $$X^\flat(Y) = g(X,Y) = X^i Y^j g_{ij}$$ so it follows that $$(X^\flat)_i = g_{ij} X^j$$


Inverse Metric


We can define a $(2,0)-$tensor $\Gamma$ which acts like a metric on covectors rather than vectors. Its defining characteristic would be the fact that


$$\Gamma(X^\flat,Y^\flat) = g(X,Y)$$ Therefore, $$(X^\flat)_m (Y^\flat)_n \Gamma^{mn} = X^i Y^j g_{mi}g_{nj}\Gamma^{mn} = X^i Y^j g_{ij}$$


It follows that the components $\Gamma^{ij}$ are the components of the inverse of the metric tensor written as a matrix. We sometimes write $\Gamma = g^{-1}$, despite the fact that the inverse of a $(2,0)-$tensor is not a meaningful notion. We also often write the components of $\Gamma$ as $g^{ij}$ - that is, the components of the metric tensor with "raised indices." This is a fairly severe abuse of notation, but such is life.


Sharp


Now that the inverse metric has been defined, we can define a sharp operation which takes a covector $\omega$ to a vector $\omega^\sharp$: $$ \omega^\sharp \equiv \Gamma(\omega,\bullet)$$


Musical Isomorphism



Notice that because of our previous definitions, $$(X^\flat)^\sharp = X$$ and $$(\omega^\sharp)^\flat = \omega$$ This correspondence is often called the musical isomorphism.


Differential Forms


A differential $k$-form is totally asymmetric $(0,k)-$tensor. Functions on $\mathcal{M}$ are 0-forms, while covectors are 1-forms. The asymmetry condition starts to be important with 2-forms, which are spanned by the basis $$\epsilon^i \wedge \epsilon^j$$. If $\mathcal{M}$ has dimension 2, then we could have



  • 0-forms: $f$

  • 1-forms: $\alpha = \alpha_1 \epsilon^1 + \alpha_2 \epsilon^2$

  • 2-forms: $\beta = \beta_{12} (\epsilon^1\wedge \epsilon^2)$


where $$\epsilon^1\wedge\epsilon^2 = -\epsilon^2\wedge\epsilon^1$$


No higher forms are possible (or rather, all higher forms are trivially 0) because of the asymmetry condition.



Hodge Star


Notice that on a manifold of dimension $d$, the set of $k-$forms has dimension ${d\choose{k}}$. Therefore, we can set up a correspondence between $k-$forms and $(d-k)-$forms. We call this operation $\star$ - the Hodge star.


In our example of a 2D manifold, the Hodge star takes a 0-form $f$ and returns a 2-form $$\star f = f (\epsilon^1 \wedge \epsilon^2)$$ and vice-versa. The action of the Hodge star on a 1-form is the identity operation.


For a 3D manifold, the Hodge star takes a 0-form $f$ and returns a 3-form $$\star f = f (\epsilon^1\wedge\epsilon^2\wedge\epsilon^3)$$ and vice-versa. It takes a 1-form $$\alpha = a \epsilon^1 + b\epsilon^2 + c\epsilon^3$$ and returns a 2-form $$\star \alpha = a (\epsilon^2\wedge\epsilon^3) + b (\epsilon^3 \wedge \epsilon^1) + c (\epsilon^1 \wedge \epsilon^2)$$ and vice-versa.


So on and so forth.


Exterior Derivative


We can define an operation called exterior differentiation which takes a $k-$form $\alpha$ and returns a $(k+1)-$form $d(\alpha)$. It is defined by the following properties:



  • $df=(\partial_i f)\epsilon^i$ for all smooth functions $f$, and

  • $d(d(f))\equiv d^2(f) = 0$ for all smooth functions $f$, and


  • $d(\alpha \wedge \beta) = d\alpha \wedge \beta + (-1)^p \alpha \wedge d\beta$ where $\alpha$ is a $p-$form and $\beta$ is a $k-$form ($k$ is unspecified)


In our 2D example, given a 0-form $f$, we have that $$d(f) = (\partial_1 f)\epsilon^1 + (\partial_2 f)\epsilon^2$$ and given a 1-form $$\alpha = \alpha_1 \epsilon^1 + \alpha_2 \epsilon^2$$ we have that $$d(\alpha) = (\partial_2 \alpha_1)\epsilon^2\wedge\epsilon^1 + (\partial_1 \alpha_2) \epsilon^1\wedge\epsilon^2=(\partial_1 \alpha_2 - \partial_2 \alpha_1)\epsilon^1\wedge\epsilon^2$$


Notice that $$d(d(\alpha))=0$$ This is no coincidence - $$d^2\alpha = 0$$ for all $k-$forms $\alpha$.


Grad,Curl,Div


We now define the operations $grad$ (which maps functions to vector fields), $curl$ (which maps vector fields to vector fields), and $div$ (which maps vector fields to functions). Note that we restrict our attention to manifolds of dimension 3.


Given a function (aka a $(0,0)-$tensor, or a $0-$form) $f$



  • $df$ is a 1-form (a covector field), and

  • $(df)^\sharp$ is a vector field



so we let $$grad(f) = (df)^\sharp$$


Given a vector field $X$,



  • $X^\flat$ is a covector field (a 1-form), and

  • $d(X^\flat)$ is a 2-form, and

  • $\star d(X^\flat)$ is a 1-form (a covector field), and

  • $[\star d(X^\flat)]^\sharp$ is a vector field


so we let $$curl(X) = [\star d(X^\flat)]^\sharp$$



Lastly, given a vector field $X$,



  • $X^\flat$ is a covector field (a 1-form), and

  • $\star X^\flat$ is a 2-form, and

  • $d(\star X^\flat)$ is a 3-form, and

  • $\star d(\star X^\flat)$ is a 0-form (a function)


so we let $$div(X) = \star d(\star X^\flat)$$


We have therefore defined the standard vector calculus operations in the language of differential forms/differential geometry. Notice that it follows immediately that


$$curl(grad(f)) = [\star d([df^\sharp]^\flat)]^\sharp = [\star d^2 f]^\sharp = 0$$ and $$div(curl(f)) = 0$$ (that one is messy due to the symbols we used, but it's pretty straightforward).



Tuesday 23 July 2019

condensed matter - Why does Fermi Level change due to change in donor atom concentration?


Suppose I have a n-type semiconductor whose fermi-level lies (say) 0.2 eV below the conduction band. Why would this level change if I changed the doping by making the donor concentration (say) 4 times the original value?


And what does it even mean? Does this mean that chemical potential also changes ? (I guess it, probably, does but what it means?)


Also could we estimate how would it change in some approximate way?




acoustics - Air oscillation at open window of a moving car


When driving a car with an open window one can hear (and feel) oscillations of air at the threshold of the open window. I used to think the open window and the car interior were forming a Helmholtz oscillator, but then noticed that the frequency of the vibration doesn't change when I change scroll the window up or down, from wide open to 3/4 closed. The amplitude of vibration changes, but the frequency change I don't notice. Thus the two-part question: what causes air to vibrate in the open window? And why doesn't the frequency change when I open the window further?




quantum mechanics - Probability and double slit


if a beam of identical particles at random distances from each other (or exactly 1/2 lambda between each other) travelling with the same v towards a double sllit do not interfere with each others wave function, so that the wave function of each particle upon reaching the double slit is always lambda / p, thus producing a predictable interference pattern,


how come the wave function of the region BETWEEN the slits DOES interact with the wavefunction of each particle so as to 'block' (or greatly reduce) it so that each particle's wave function can interfere with ITSELF via the two slits?


conversely, since the wavefunction of one particle (the inter-slit 'substance') can and seemingly does affect the wavefunction of another 'particle' (that of the approaching particle), why don't the particles in a beam interfere with each other so as to randomoly destroy any assignable wavelength to them?


if the inter-slit region had no affect on the wavefunctions of the approaching particles, the diffraction grating / double slit apparatus would be entirely transparent to the beam and there would be no interference at all.


The probability function is an entirely mathematical construct, and yet how it evolves over space must be dependent on whether there is any 'matter' in the region through which it passes. Is there something like a "damping factor" of freespace, for the probability function, like electrical permitivity of freespace?




gravity - The trajectory of a projectile launched from a hilltop


Here is the problem:


A boy stands at the peak of a hill which slopes downward uniformly at angle $\phi$. At what angle $\theta$ from the horizontal should he throw a rock so that it has the greatest range?



I realize that the same question is posted here: https://physics.stackexchange.com/questions/24235/trajectory-of-projectile-thrown-downhill, but I have some questions that were not answered in that thread:



  1. Can the problem be solved without a rotation of the coordinate system? If so, how?

  2. I tried to solve the problem using a rotated coordinate system, but cannot figure out how to finish it (see the work given below).


Here is what I have so far:



  1. We set up the coordinate system so that the positive $x$ axis coincides with the downward slope of the hill. This simplifies the problem by allowing us to easily relate $\phi$ and $\theta$, through the relation $\alpha = \phi + \theta$.

  2. $v_{0x} = v_0 \cos\alpha$

  3. $v_{0y} = v_0 \sin\alpha$


  4. $a_x=-g \cos(\phi-\frac{\pi}{2})=-g \cos (-(\frac{\pi}{2}-\phi))= g\cos(\frac{\pi}{2}-\phi)=g\sin\phi$

  5. $a_y=-g \sin(\phi-\frac{\pi}{2})=-g \sin (-(\frac{\pi}{2}-\phi))= -g\sin(\frac{\pi}{2}-\phi)=-g\cos\phi$

  6. $v_x=v_{0x}+\int_{0}^{t}{a_x(t \prime)dt \prime}=v_0 \cos \alpha+\int_{0}^{t}{(g\sin\phi) dt \prime} =v_0 \cos \alpha + t(g\sin\phi)$

  7. $x=x_0 + \int_0^t{v_x(t\prime) dt\prime} = \int_0^t{(v_0 \cos \alpha + t\prime(g\sin\phi)) dt\prime}=t(v_0 \cos \alpha) + \frac{1}{2}t^2(g \sin \phi)$

  8. $v_y=v_{0y}+\int_{0}^{t}{a_y(t \prime)dt \prime}=v_0 \sin \alpha+\int_{0}^{t}{(-g\cos\phi) dt \prime} =v_0 \sin \alpha - t(g\cos\phi)$

  9. $y=y_0 + \int_0^t{v_y(t\prime) dt\prime} = \int_0^t{(v_0 \sin \alpha - t\prime(g\cos\phi)) dt\prime}=t(v_0 \sin \alpha) - \frac{1}{2}t^2(g \cos \phi)$

  10. To find the flight time of the projectile, we find the time at which its trajectory intersects the ground (in this case, the $x$ axis), by setting $y=0$ and solving for $t$. $$y=t(v_0 \sin \alpha) - \frac{1}{2}t^2(g \cos \phi)=0$$ $$v_0 \sin \alpha = \frac{1}{2}t(g \cos \phi)$$ $$t=\frac{2v_0 \sin\alpha}{g \cos \phi}$$

  11. Substituting $t$ into the equation for $x$ gives us the distance traveled by the projectile as a function of the angles $\alpha$ and $\phi$.$$x=t(v_0 \cos \alpha) + \frac{1}{2}t^2(g \sin \phi)$$ $$x=(\frac{2v_0 \sin\alpha}{g \cos \phi})(v_0 \cos \alpha) + \frac{1}{2}(\frac{2v_0 \sin\alpha}{g \cos \phi})^2(g \sin \phi)$$ $$x=\frac{2v_0^2}{g \cos \phi}(\sin \alpha \cos \alpha)+\frac{2v_0^2}{g \cos \phi}(\sin^2\alpha \frac{\sin\phi}{\cos\phi})$$ $$x=\frac{2v_0^2}{g \cos \phi}(\sin\alpha\cos\alpha+\sin^2\alpha\tan\phi)$$

  12. I noticed that the solution in the other thread proceeds from here by differentiating $x$ with respect to $\alpha$, holding $\phi$ constant, which gives $$\frac{dx}{d\alpha}=\frac{2v_0^2}{g \cos \phi}(\frac{d}{d\alpha}(\frac{1}{2}(\sin(2\alpha)+\sin^2\alpha\tan\phi))$$ $$\frac{dx}{d\alpha}=\frac{2v_0^2}{g \cos \phi}(\cos(2\alpha)+2\sin\alpha\cos\alpha\tan\phi)$$ $$\frac{dx}{d\alpha}=\frac{2v_0^2}{g \cos \phi}(\cos(2\alpha)+\sin(2\alpha)\tan\phi)$$ This equation allows us to examine how $x$ changes with respect to $\alpha$. We see that $x$ increases as $\alpha$ increases, up to a certain point, and then decreases as $\alpha$ increases past this value. This means that the graph of $x$ has a relative maximum at the value of $\alpha$ which produces the maximum range.

  13. We want to find the value of $\alpha$ that results in the maximum range of the projectile. In other words, we must determine the value of $\alpha$ for which the graph of $x$ has a relative maximum. We achieve this by setting $$\frac{dx}{d\alpha}=0=\frac{2v_0^2}{g \cos \phi}(\cos(2\alpha)+\sin(2\alpha)\tan\phi)$$ Dividing each side by $\frac{2v_0^2}{g \cos \phi}$ produces $$\cos(2\alpha)+\sin(2\alpha)\tan\phi=0$$ Here is where I get lost. It seems like this should be the easy part, because the only thing left to do is solve the above equation for $\alpha$, but I don't know how to do it. Could someone please explain this part to me?



Additionally, I would like to know whether the problem can be solved without rotating the coordinate system. I originally set out to solve it using the standard rectangular coordinate system, but got bogged down in some equations that seemed to lead nowhere. Thanks for your help.




Monday 22 July 2019

quantum field theory - If renormalization scale is arbitrary, why do we care about running couplings?


For the bounty please verify the following reasoning


[copied from comment below]


Ah right, so the idea is that overall observable quantities must be independent of the renormalization scale. But at each order in perturbation theory the result can depend on renormalization scale, right? And they do so in exactly the right way to invalidate the use of a perturbation series when external momenta are of the wrong order. This is because the external momenta get caught up in loop integrals due to momenta conservation at vertices and the renormalization group equation comes along and means the right things cancel.


[original question]



In Brief


People say that perturbation theory breaks down when the couplings run to high values. But given that this running depends on an arbitrary mass scale, how is this argument logical?!


A Longer Exposition (TL;DR)


It's obvious that Feynman diagram techniques work best when the coupling constant is small, since then you can neglect higher order terms. But various sources (e.g. Peskin and Schroeder) make claims about running couplings that seem incomplete to me.


I often read sentences like



  • if the renormalized coupling is small at low energy, then Feynman diagrams are good in that region

  • for asymptotically free theories, Feynman diagrams are good for high energy calculations


I understand exactly why these are true. But surely something much more general is true, namely




  • if the coupling constant is small at any renormalization scale the Feynman diagrams are good for calculations


My reasoning is as follows. Observable quantities are completely independent of the running of the coupling, so you can just choose your scale appropriately so that the coupling is small for the expansion. Hey presto, you can use Feynman diagrams.


Is this true? I strongly expect the answer to be no but I can't convince myself why. Here's my attempt at a self-rebuttal.



  • my argument above is incorrect, because I've assumed there's only a single coupling. In reality there's contributions from "irrelevant" terms at higher energies whose couplings can't be fixed from low energy observations.


This leads me to hypothesise that the following is true (would you agree?)




  • if the coupling constant is small at any renormalization scale above the scale of your observations then Feynman diagrams are good


This seems more plausible to me, but it would mean that Feynman diagrams would be good for low energy strong interaction processes for example. This feels wrong is a sense because the renormalized coupling is large there. But then again the renormalization scale is arbitrary, so maybe that doesn't matter.


Many thanks in advance!




spacetime - How can the permittivity & permeability of the vacuum work as a viscosity term for the speed of light?


I've often read that the speed of light more about the speed of events than light specifically. I wonder if this implies that spacetime has some base viscosity, or mathematically similar mechanic.


To restate, I'm not really asking if there is a literal, physical viscosity. For a simulation, I am considering whether or not I can relate the cosmological constant to a viscosity property with some special treatment in a similar algorithm to what's used for handling refraction depending on type/density of materials.


The main goal is to get a pretty accurate speed of light as an emergent property rather than a hardcoded limit.



Answer




It is the permittivity and permeability of vacuum that determine the speed of light. Those two come from how strong an electric field is created by charge in vacuum and similarly a magnetic field. C is the inverse square root of those two multiplied. Nowadays those are considered simply part of defining the units one uses, with c more basic.


See in Wikipedia at https://en.m.wikipedia.org/wiki/Maxwell%27s_equations


In materials those are typically larger and the equivalent effective speed of light then goes down.


If ones of those were 0, or both, which makes no sense, infinite electric or magnetic fields would be induced in an electromagnetic wave and it would propagate infinitely fast, which of course makes no physical sense, only that's what Maxwells equations would lead to.


So in essence the speed of light is the best measure of how resistant the vacuum is to creating electromagnetic waves. Maybe this could represent your undefined viscosity, but it won't do anything to explain why c is what it is, and the fact that it is finite. It's that if it was infinite all effects in the universe would be instanataneous, there would be no localization.


special relativity - Explanation for a much simpler version of the twin paradox?


I have seen the classical twin paradox before. It uses a twin stationary on Earth and the other traveling away and back. I have seen many contradictory solutions for it, some use general relativity, others use special relativity, either way, I am still troubled by it. They always try to break the symmetry through the traveling twin's acceleration and deceleration, but never quite succeed.


So, let's do away with the classical twin paradox and let's explain a much simpler, perfectly symmetrical version of it where both twins are moving towards each other.


So imagine we have Twin A in a spaceship, and Twin B in another, a light year apart from each other at the beginning of the experiment. They both start traveling at the same speed towards each other when the first light of one twin since the beginning of the experiment reaches the other, so they effectively start moving one year into the experiment.


If I understand relativity properly:




  • From Twin A's frame of reference, he's stationary and Twin B is moving at a constant speed towards him, therefore, because of time dilation, Twin B's clock is ticking slower.

  • From Twin B's frame of reference, he's stationary and Twin A is moving at a constant speed towards him, therefore, because of time dilation, Twin A's clock is ticking slower.


Regardless of what their observations might be because of the Kepler effect and what not, time dilation dictates that a moving clock will absolutely tick slower than a stationary one. So, because Twin B is moving relative to Twin A, Twin B's clock is absolutely ticking slower than Twin A's. The same is supposed to be true the other way around. This is obviously a contradiction.




special relativity - Lorentz transformation of classical Klein–Gordon field


I'm trying to see that the invariance of the Klein–Gordon field implies that the Fourier coefficients $a(\mathbf{k})$ transform like scalars: $a'(\Lambda\mathbf{k})=a(\mathbf{k}).$


Starting from the mode expansion of the field


$$\phi'(x)=\phi(\Lambda^{-1}x)= \int \frac{d^{3}k}{(2\pi)^{3}2E_{k}} \left( e^{-ik\cdot \Lambda^{-1}x}a(\mathbf{k}) +e^{ik\cdot \Lambda^{-1}x}b^{*}(\mathbf{k}) \right),$$


it's easy to see that it equals



$$\int \frac{d^{3}k}{(2\pi)^{3}2E_{k}} \left( e^{-i(\Lambda k)\cdot x}a(\mathbf{k}) +e^{i(\Lambda k)\cdot x}b^{*}(\mathbf{k}) \right).$$


when using the property $\Lambda^{-1}=\eta\Lambda^{T}\eta$. Then doing a change of variable $\tilde{k}=\Lambda k$ and considering orthochronous transformations so that the Jacobian is 1, then I get the wanted result ($a'(\Lambda\mathbf{k})=a(\mathbf{k})$) when comparing with the original mode expansion. However, this is not quite right as I would have to justify that $E_\tilde{k}=E_k$ but I can't see how.



Answer



The important insight is that it's actually the whole combination $$ \frac{d^3 k}{2(2\pi)^3 E_\mathbf k}, \qquad E_\mathbf{k} = \sqrt{\mathbf k^2 + m^2} $$ that forms a Lorentz-invariant measure. To see this, note that if we define $k= (k^0, \mathbf k)$ and use the identity $$ \delta(f(x)) = \sum_{\{x_i:f(x_i) = 0\}} \frac{\delta(x-x_i)}{|f'(x_i)|} $$ then we get $$ \delta(k^2 - m^2)=\frac{\delta(k^0 - \sqrt{\mathbf k^2+m^2})}{2\sqrt{\mathbf k^2+m^2}} + \frac{\delta(k^0 + \sqrt{\mathbf k^2+m^2})}{2\sqrt{\mathbf k^2+m^2}} $$ so the original measure can be rewritten as $$ \frac{d^3 k}{2(2\pi)^3 E_\mathbf k}=\frac{d^3k\,d k^0}{2(2\pi)^3 k^0}\delta(k^0 - \sqrt{\mathbf k^2+m^2}) = \frac{d^4k}{(2\pi)^3}\delta(k^2-m^2)\theta(k^0) $$ which is manifestly Lorentz invariant for proper, orthochronous Lorentz transformations. The rest of your manipulations go through unscathed, and you get the result you want!


Hope that helps!


Cheers!


soft question - How to learn physics effectively and efficiently



How do you effectively study physics? How does one read a physics book instead or just staring at it for hours?


(Apologies in advance if the question is ill-posed or too subjective in its current form to meet the requirements of the FAQ; I'd certainly appreciate any suggestions for its modification if need be.)




homework and exercises - How to find the total current supplied to the circuit?



Recently, I came across a question based on finding electric current of a circuit. Here's the image...



I know, by using the formula $I=V/R$, we can easily calculate the current as $V$ is given and $R$ can be calculated from the diagram. In the book (from where I got the question),



Solve the $R$ (net) by combining the $6 \Omega$ and $2 \Omega$ resistances in parallel, and with both, $1.5 \Omega$ in series and whole parallel with $3 \Omega$.



I didn't get the logic they used. First, I thought of keeping 6, 2 and 1.5 ohm resistors in parallel and with all, the 3 ohm in series. But, that didn't work. Can someone please help me?




Sunday 21 July 2019

quantum mechanics - Considering an arbitrary wavefunction for a free particle, are all normalizable functions valid?


One can show that a possible solution to a wavefunction with constantly zero potential is equal to, only considering the spacial piece:


$$\psi(x) = \int_{-\infty}^{\infty} A(k) \ e^{ikx} dk$$


This is partially due to the fact that, since there are no boundary conditions applying a rule to $k$, such as $k = \frac{n \pi}{L}$ like in the case of the infinite square well. That merely explains why a sum of eigenstates cannot represent an arbitrary ket vector. This is, however, mainly due to the fact that a sum is not only unnecessary but inoperable, as a Fourier sum without boundary conditions like the infinite square well will be periodic throughout space and hence will not be normalizable and have weird implications on its position and momentum uncertainty I would think. This is why a Fourier integral is necessary.


However, if I am to postulate that $A(k)$ is the Fourier transform of $\psi(x)$, then could I not contend that this is just the Fourier transform of some arbitrary function $f(x)$ in disguise, and the solution to Schrodinger's equation given constant zero potential? In this case, since $f(x)$ is arbitrary, any function would be a solution given this case, as long as it is normalizable. Why or why not?




Is there a prediction of quantum mechanics that could be construed as representing an "energy-time uncertainty relation?"



As the title suggests. Is there a prediction of quantum mechanics that could be construed as representing an "energy-time uncertainty relation?" Does there exist any reference to such a prediction, or is it obviously false on some level?



Answer



There is a 1961 paper by Aharonov and Bohm on this subject, in which there is defined, among other things, a characteristic time for an operator's expectation value to deviate significantly, measured by the initial dispersion in that operator. This result is essentially a theorem we will prove here (Theorem 2).




Let $\mathcal{S}$ be a Hilbert space, $A$ a (bounded) Hermitian operator on $\mathcal{S}$, and $\psi$ a unit vector in $\mathcal{S}$. By the dispersion of $A$ $($relative to the state $\psi$$)$ we mean the number$$\Delta_\psi A = \left( \left\langle \psi\,\left|\,A^2\,\right|\,\psi \right\rangle - \left\langle \psi\,\left|\,A\,\right|\,\psi\right\rangle^2 \right)^{{1\over2}} = \left( \left\langle \psi\,\left|\,\left(A - \langle \psi\,|\,A\,|\,\psi\rangle I\right)^2\,\right|\,\psi \right\rangle \right)^{{1\over2}}.$$So, for example, the dispersion is nonnegative, and vanishes if and only if $\psi$ is an eigenvector of $A$.


Since $\Delta A$ is merely a number – and not a Hermitian operator on the Hilbert space – we cannot of course make an observation via $\Delta A$. But we can do the next best thing. Introduce two ensembles of copies of system $\mathcal{S}$, each copy in initial state $\psi$. Allow an instrument to observe in succession the systems in the first ensemble, via $A$; and a second instrument for the second ensemble, via $A^2$. Finally, introduce a third instrument that observes the first two instruments via an appropriate operator to "compute $\Delta A$. In this sense, then, the number $\Delta A$ has direct physical significance.



Theorem 1. Let $A$ and $B$ be $($bounded$)$ Hermitian operators on Hilbert space $\mathcal{S}$, and $\psi$ a unit vector. Then $$\Delta_\psi A \Delta_\psi B \ge {1\over2}\left|\langle \psi\,\left|\,[A, B]\,\right|\,\psi\rangle\right|.$$


Proof. This is the standard uncertainty relations for noncommuting observables in quantum mechanics.


$\tag*{$\square$}$


But what of the "energy-time uncertainty relation?"


Theorem 2. Let $H$ be a $($bounded$)$ Hermitian operator on Hilbert space $\mathcal{S}$, and $\psi$ a unit vector. Set $P = I - |\psi\rangle\langle\psi|$, the projection operator orthogonal to $\psi$, and $\psi_t = e^{{{-i}\over{\hbar}} Ht\psi}$. Then, for every number $\Delta t \ge 0$, we have$$\Delta_\psi H \Delta t \ge \hbar \left|\langle \psi_{\Delta t}\, |\, P \,|\, \psi_{\Delta t}\rangle\right|^{{1\over2}}.$$


Proof. Set, for each $t \ge 0$, $\alpha(t) = \langle \psi_t \,|\,P\,|\,\psi_t\rangle$. Then we have $$\left|{{d\alpha}\over{dt}}\right| = \left|\left\langle \psi_t\, \left|\, {i\over\hbar} [H, P]\,\right|\, \psi_t\right\rangle\right| = {2\over\hbar} \Delta_{\psi_t} H \Delta_{\psi_t} P,$$where we used Theorem 1 in the second step. but $\Delta_{\psi_t}H = \Delta_\psi H$, and $$\Delta_{\psi_t} P = \left( \left\langle \psi_t\,\left|\,P^2\,\right|\,\psi_t \right\rangle - \left\langle \psi_t\,\left|\,P\,\right|\,\psi_t\right\rangle^2 \right)^{{1\over2}} = \left(\alpha - \alpha^2\right)^{1\over2} \le \alpha^{1\over2}.$$Substituting, we obtain $$\left|{{d\alpha}\over{dt}}\right| \le \left({2\over\hbar}\right) \left(\Delta_\psi H\right) \alpha^{1\over2}.$$Dividing this inequality by $\alpha^{1\over2}$, integrating from $t = 0$ to $t = \Delta t$, and using that $\alpha(0) = 0$, the result follows.


$\tag*{$\square$}$


To apply this theorem to physical problems, we choose for $H$, of course, the Hamiltonian of the system. Then $\psi_t$ is the solution of Schrödinger's equation with initial $(t = 0)$ state $\psi$. Further, $\Delta_{\psi} H$ is the energy dispersion relative to this initial state. We may interpret $\left|\left\langle \psi_t\,\left|\,P\,\right|\,\psi_t\right\rangle\right|^{1\over2} = \|P\psi_t\|$ as a measure of how much the state $\psi_t$ differs from the initial state $\psi$. Thus, this expression vanishes for $t = 0$ $($when $\psi_t = \psi$$)$, and grows from zero only as $\psi_t$ acquires a component orthogonal to $\psi$. So, the theorem states, roughly the following: "In order to obtain an evolved state $(\psi_{\Delta t})$ appreciably different from the initial state $(\psi)$, you must wait sufficiently long $(\Delta t)$, and have sufficient dispersion in the initial state $(\Delta_\psi H)$ that the product of these two is greater than the order of $\hbar$." Of course, the theorem, with these choices, does make a testable prediction of quantum mechanics, using suitable ensembles as described earlier.






One cannot, in time $\Delta t$, measure the energy of a system within error $\Delta E$ unless $\Delta E\Delta t \ge \hbar$.



I do not know what this statement means, for I do not know what experiment is being contemplated. The part "...measure the energy of a system within error..." suggests the idea that quantum systems have some "actual energy." But, according to quantum mechanics, they do not. What they "have" is a ray in their Hilbert space of states, while the energy is an operator on this Hilbert space. (Perhaps this idea is a holdover from classical mechanics, in which systems do "have an energy," for there the energy is a function on the space of classical states.) Further, even if we thought of quantum systems as having some true energy, it is not clear how we are supposed to acquire the information as to the discrepency between this true energy and our measured value.



On an ensemble of systems, all in initial state $\psi$, let there be made a determination, carried out in time $\Delta t$, of the dispersion of the Hamiltonian, $\Delta_\psi H$. Then $\Delta_\psi H \Delta t \ge \hbar$. $($See earlier.$)$



This is intended as a clarified version of the first statement. Now, it is claimed, the experiment is more or less clear. But, unfortunately, this statement is false. For fixed $\psi$ and $H$, I see no obstacle to making this determination in an arbitrarily small time $\Delta t$. We merely turn the interaction on and then off quickly.



Consider an ensemble of systems, all in initial state $\psi$, and an ensemble of instruments, each of which will make an observation on a corresponding system via the Hamiltonian $H$, in time $\Delta t$. Introduce observable $E$ on the instrument Hilbert space corresponding to "the reading of the instrument." Determine the dispersion of $E$, $\Delta E$. Then $\Delta E \Delta t \ge \hbar$.




This is intended as a second possible version of the first statement. It is perhaps more in the spirit of that statement, for now we determine the dispersion of the "energy readings of the instruments," and not of the system's energy. But this statement is also false in quantum mechanics.



A measurement of the energy of a system, made in time $\Delta t$, will disturb the energy of that system by at least amount $\Delta E$, where $\Delta E \Delta t \ge \hbar$.



I do not know what this system means, for I do not know what experiment is contemplated. Again, the part about "...disturb the energy of that system..." suggests that systems in quantum mechanics "have a true energy." But even if they did, it is not clear how we are supposed to acquire the knowledge of by how much that energy was disturbed.



On an ensemble of systems, all in initial state $\psi$, let there be made an observation via the Hamiltonian of each system, in time $\Delta t$. Subsequent to this, let there be made on this ensemble a determination of the dispersion in the Hamiltonian, $\Delta H$. $($See earlier.$)$ Then $\Delta H \Delta t \ge \hbar$.



This is intended as a clarified version of the statement above. Now, we make an energy observation on each system in the ensemble, and then, for the ensemble taken as a whole, we determine its Hamiltonian-dispersion. The idea is that this "subsequent Hamiltonian-dispersion" will be at least so large, by virtue of the earlier observation, via the Hamiltonian, at time $\Delta t$. But this last statement is false. For instance, suppose that the initial state $\psi$ were a Hamiltonian eigenstate. Then the result of the first observation would leave each system in that eigenstate; whence the determination of $\Delta H$ would yield zero. Clearly, then, we would in this case violate the assertion above.





I do not know whether there are any statements that are both meaningful and true, along the lines above. But there does seem to be at least one statement that does appear to reflect "energy-time uncertainty," i.e. Theorem 2.


Another paper that is relevant is one by Sorkin $($Foundations of Phys, 9, 123 $($1979$)$$)$. While it is true that one can time-Fourier-transform the wave function, and thereby derive an "uncertainty relation" between $\Delta t$ and $\Delta \omega$ $($frequency$)$, it is not clear what that means physically. I could see someone quoting the old Einstein-Bohr argument, and in particular the example of the box hanging by a spring, emitting a photon under the control of a shutter, but I have never really understood what that debate was about.


electricity - Why does lightning emit light?


What exactly is causing the electric discharge coming from the clouds to emit light while traveling through the air. I've read and thought about it a little but with my current knowledge I cant really figure what is happening at a subatomic level. In other words, what causes the emission of photons during lightning. What I want to know is, is there some sort of collisions taking place that produces the photons we see, or is it some other interaction that is causing it?. I don't know too much about particle physics, I've only read a couple of pages in the Introduction to Elementary Particles by Griffihts. So if you could keep the answer simple then that would be great.




Saturday 20 July 2019

homework and exercises - Pressure just before the hole in a draining tank


I get extremely confused in fluid dynamics problem where the pressure in some points is required.


Consider the ideal fluid in the tank in the following picture. I would like to understand what is the pressure of the fluid in the point a. under the condition of steady flow ($A_1 \gg A_2$). The point a. is located immediately before the start of the hole, so it is still contained in the tank. enter image description here


My guess: In such problems, where the fluid in the tank is considered to not move at all, I can treat the tank as a static fluid, and the fluid in the tube as a fluid in motion. That means that the pressure in $a.$ is $P_a=P_1+\rho g h$, while the pressure in $b.$, from Bernoulli and continuity equation, is $P_b=P_2$, since $v_b=v_2$ and $h_b=h_2$.


Of course there are very big approximations here, but is this the correct way to think about the problem in the limits of steady flow?


The other option would be that the velocity in $a.$ is equal to $v_b$, but in that case it would be $P_a=P_b=P_2$, from Bernoulli equation, which is even more strange.





If I meet the same problem with a fluid which is not ideal, still incompressible but with viscosity $\eta \neq 0$, would anything change? Would the consideration $P_a=\rho g h +P_1$ still be valid?



Answer



In your approximation there is a velocity discontinuity where the pipe joins the container. Assuming that is a good approximation then you are correct and $P_a=P_b+\rho v^2/2$. In real life though, the pressure will change smoothly because the closer you get to the pipe, the larger the effect of the motion of the water inside the tank.


Can "particle" waves break as ocean waves do?


I have heard about electrons surfing on wake fields which got me thinking. Are there analogs to reefs for these waves and can these waves break as ocean waves do when they hit a reef?




Friday 19 July 2019

general relativity - Photon pair production at relativistic speeds?


This is probably an obvious question, but I don't see it answered anywhere.


Imagine we have an object in the Universe that is traveling, relative to the Milkyway at about .99999999999999999999999999c and I realize that's not possible, but imagine.


Speed is relative, so to the object, the Milkyway is flying past it and to us, the object is flying past us. That's all good and relative.



But the photons that our superfast object encounters are gonzo blue-shifted, and relativity says that's OK too. But lets say the object is traveling so fast, the photons it encounters are energetic enough to spontaneously undergo pair production. That seems difficult to explain. A relatively low energy visible light photon to our perception won't split into an Electron and a Positron, but a very energetic photon can, but it's the same photon just observed by 2 different reference frames?


I understand that observers won't agree on when things happen or on energy but the two observers should agree on events that take place and pair production for one but not the other seems impossible.


Is there a simple layman's way that this can be explained?



Answer



Yes, there is a simple explanation! The pair production process $\gamma \to e^+ + e^-$ is forbidden by energy-momentum conservation, so it doesn't happen in either frame.


One way to see this is that, as a massless object, the photon has "the most possible momentum for its energy", as it doesn't have the extra rest mass-energy. So if you try to make the momentum in this equation balance, the electron and positron will have too much energy, and vice versa.


However, a slightly modified version of your idea holds. Our universe is full of low-energy photons from the Cosmic Microwave Background. As you suggested, a very fast moving charged particle will see these photons as heavily blueshifted, and pair production is possible if the photons scatter off the object. (This is a $2 \to 2$ reaction, so the argument in the previous paragraph doesn't hold.)


The scattering steals some of the particle's energy, slightly slowing it down. This places a limit on the energy of cosmic rays, called the GZK bound.


Understanding Stagnation point in pitot fluid

What is stagnation point in fluid mechanics. At the open end of the pitot tube the velocity of the fluid becomes zero.But that should result...